Đến nội dung

Hình ảnh

Dãy số đầu năm

* * * * - 3 Bình chọn

  • Please log in to reply
Chủ đề này có 20 trả lời

#1
hxthanh

hxthanh

    Tín đồ $\sum$

  • Hiệp sỹ
  • 3921 Bài viết
Đầu xuân năm mới, xin gửi tới toàn thể members of VMF, lời chúc:
An khang thịnh vượng, vạn sự như ý, sức khỏe dồi dào, phong độ lên cao, bài nào cũng "chém"! :))
----
Có vài bài dãy số sau: Hy vọng các bạn ủng hộ :)

Bài 1:
Tìm số hạng thứ $n $ của dãy: "Thứ tự tăng dần của các số tự nhiên lẻ không chính phương"
$.\;\;\;\;\;\;\;\{u_n\}_1^\infty:\;\;\{3,5,7,11,13,15,17,19,21,23,27,...\}$

Bài 2:
  • Tìm số hạng tổng quát $u_n$
  • Tính tổng $n $ số hạng đầu tiên $\sum_{k=1}^n u_k$
của dãy $\{u_n\}$, trong các trường hợp sau:

$\text{a)}\;\;\;\;\;\;\;\{u_n\}_1^\infty:\{1,2,2,3,3,3,4,4,4,4,...\}$,......(1 số 1, 2 số 2, 3 số 3, 4 số 4,...)

$\text{b)}\;\;\;\;\;\;\;\{u_n\}_1^\infty:\{1,2,3,3,4,4,5,5,5,6,6,6,...\}$,.......(1 số 1, 1 số 2; 2 số 3, 2 số 4; 3 số 5, 3 số 6; 4 số ...)

Bài 3:
Tính tổng $S_n=\sum_{k=0}^n \left\lfloor 2\sqrt k\right\rfloor$

Bài 4:
Chứng minh rằng với $p$ là số nguyên tố bất kỳ ta có:
$\sum_{k=0}^{(p-1)(p-2)} \left\lfloor\sqrt[3]{kp}\right\rfloor=\dfrac{(p-1)(p-2)(3p-5)}{4}$
------
Phần Nguyên: Định nghĩa đơn giản, tính chất cũng đơn giản, nhưng sao ứng dụng và bài tập thì... chẳng đơn giản tẹo nào :P

#2
hxthanh

hxthanh

    Tín đồ $\sum$

  • Hiệp sỹ
  • 3921 Bài viết
Mấy bài này, tuy hơi khó một chút, nhưng không đến nỗi không nghĩ ra nổi cách làm.
Các bạn hãy thử làm xem, rất thú vị đấy!

Ai làm bài 1 giơ tay!
(Đếm xem từ 1 đến U_n thì có bao nhiêu số chính phương lẻ, loại đi, lùi chỉ số của U_n về tương ứng)
Bạn nào xung phong chém bài 2 câu 1.a nào?
(Xét U_n ở 1 nhóm các số hạng giống nhau có giá trị m, suy ra có m-1 nhóm số hạng nhỏ hơn...)

Topic ế ẩm quá :(

#3
hxthanh

hxthanh

    Tín đồ $\sum$

  • Hiệp sỹ
  • 3921 Bài viết
GIẢI MẪU CHO CÁC BẠN BÀI NÀY

Bài 1:
Tìm số hạng thứ $n$ của dãy: "Thứ tự tăng dần của các số tự nhiên lẻ không chính phương"
$\{u_n\}_1^\infty:\;\;\{3,5,7,11,13,15,17,19,21,23,27,...\}$
Xét dãy các số tự nhiên lẻ:
$\{d_n\}_1^\infty:\;\{1,3,5,7,9,11,13,...\},\;(d_n=2n−1)$
Dễ thấy $u_n=d_n+2k$ Trong đó $k$ là số các số chính phương lẻ nhỏ hơn $u_n$
(Loại đi các số chính phương lẻ thì $\{d_n\}\equiv\{u_n\}$. Loại $1$ số thì $d_i$ tăng $2$,... loại $k$ số thì $d_i$ tăng $2k$)
Xét khoảng cách giữa hai số chính phương lẻ liên tiếp
$(2i+1)^2−(2i−1)^2−1=8i−1$ (có $8i−1$ số tự nhiên)
Suy ra trong khoảng đó có $4i−1$ số tự nhiên lẻ
Giả sử $u_n$ nằm giữa hai số chính phương lẻ liên tiếp là $(2k−1)^2$ và $(2k+1)^2$ (số chính phương lẻ thứ $k$ và thứ $k+1$)
Như vậy có $k$ số chính phương lẻ nhỏ hơn $u_n$ Suy ra có $k−1$ khoảng cách giữa các số chính phương đó
Đếm các số hạng của $\{u_n\}$ có giá trị nhỏ hơn $(2k−1)^2$ ta có:
$\sum_{i=1}^{k-1} (4i-1)=2k^2-3k+1$
Suy ra chỉ số $n$ phải thỏa mãn $2k^2−3k+2\le n\le 2k^2+k$

$\Rightarrow \dfrac{\sqrt{8n+1}-1}{4} \le k \le \dfrac{\sqrt{8n-7}+3}{4}$

Vì $\dfrac{\sqrt{8n-7}+3}{4}-1<\dfrac{\sqrt{8n+1}-1}{4} \le k \le \dfrac{\sqrt{8n-7}+3}{4}$

Nên: $k=\left\lfloor\dfrac{\sqrt{8n-7}+3}{4}\right\rfloor$

Kết quả cuối cùng ta có:

$\boxed{\boxed{u_n=d_n+2k=2n-1+2\left\lfloor\dfrac{\sqrt{8n-7}+3}{4}\right\rfloor}}$

#4
dark templar

dark templar

    Kael-Invoker

  • Hiệp sỹ
  • 3788 Bài viết
Em ủng hộ bài tập cho topic của anh cho :D Em chưa đủ trình để xơi mấy bài này ,mong anh thứ lỗi :P
1/Tính $H = \left\lfloor {\sum\limits_{i = 1}^k {\dfrac{1}{{\sqrt k }}} } \right\rfloor $
2/Tính $\U = \sum\limits_{i = 1}^k {\left\lfloor {\sqrt {k\left( {k + 1} \right)\left( {k + 2} \right)\left( {k + 3} \right)} }\right\rfloor } $
"Do you still... believe in me ?" Sarah Kerrigan asked Jim Raynor - Starcraft II:Heart Of The Swarm.

#5
hxthanh

hxthanh

    Tín đồ $\sum$

  • Hiệp sỹ
  • 3921 Bài viết

Em ủng hộ bài tập cho topic của anh cho :D Em chưa đủ trình để xơi mấy bài này ,mong anh thứ lỗi :P
1/Tính $H = \left\lfloor {\sum\limits_{k = 1}^n {\dfrac{1}{{\sqrt k }}} } \right\rfloor $
2/Tính $U = \sum\limits_{k = 1}^n {\left\lfloor {\sqrt {k\left( {k + 1} \right)\left( {k + 2} \right)\left( {k + 3} \right)} }\right\rfloor } $

Ok
Để anh "chém" luôn!
Bài 2 trước

$.\;\;\;U=\sum_{k=1}^n \left\lfloor {\sqrt {k\left( {k + 1} \right)\left( {k + 2} \right)\left( {k + 3} \right)} }\right\rfloor$
$\Rightarrow U=\sum_{k=1}^n \left\lfloor \sqrt{(k^2+3k+1)^2-1} \right\rfloor = \sum_{k=1}^n (k^2+3k)$
$\Rightarrow U=\dfrac{n(n+1)(2n+1)}{6}+\dfrac{3n(n+1)}{2}=\dfrac{n(n+1)(n+5)}{3}$

Bài 1: Anh nghĩ có thể dùng đến BĐT:

$\dfrac{1}{\sqrt{k+1}}<\dfrac{2}{\sqrt{k}+\sqrt{k+1}}<\dfrac{1}{\sqrt{k}}$
Hay
$\dfrac{1}{\sqrt{k+1}}<2(\sqrt{k+1}-\sqrt{k})<\dfrac{1}{\sqrt{k} }$
Để lát nữa giải tiếp (bây giờ anh đi coi đá banh! :) )
----
Đã sửa, cái tội ẩu (dark templar cũng nên xem lại kết quả nhé!)

#6
dark templar

dark templar

    Kael-Invoker

  • Hiệp sỹ
  • 3788 Bài viết

Ok
Để anh "chém" luôn!
Bài 2 trước

$.\;\;\;U=\sum_{k=1}^n \left\lfloor {\sqrt {k\left( {k + 1} \right)\left( {k + 2} \right)\left( {k + 3} \right)} }\right\rfloor$
$\Rightarrow U=\sum_{k=1}^n \left\lfloor \sqrt{(k^2+3k+1)^2-1} \right\rfloor = \sum_{k=1}^n (k^2+3k+1)(1)$
$\Rightarrow U=\dfrac{n(n+1)(2n+1)}{6}+\dfrac{3n(n+1)}{2}+n=\dfrac{n(n+2)(n+4)}{3}$

Bài 1: Anh nghĩ có thể dùng đến BĐT:

$\dfrac{1}{\sqrt{k+1}}<\dfrac{2}{\sqrt{k}+\sqrt{k+1}}<\dfrac{1}{\sqrt{k}}$
Hay
$\dfrac{1}{\sqrt{k+1}}<2(\sqrt{k+1}-\sqrt{k})<\dfrac{1}{\sqrt{k} }$
Để lát nữa giải tiếp (bây giờ anh đi coi đá banh! :) )

Bài 2 chỗ (1) ko có cộng thêm 1 đâu anh nhé :lol: nên kết quả của anh cũng khác kết quả của em .Để em trình bày cách giải của em,anh xem có chỗ nào sai ko?
Xét $\sqrt {k\left( {k + 1} \right)\left( {k + 2} \right)\left( {k + 3} \right)} = \sqrt {t\left( {t + 2} \right)} \left( {t = k\left( {k + 3} \right) > 0,t \in Z} \right) $
$t < \sqrt {t\left( {t + 2} \right)} < t + 1 \Rightarrow \left\lfloor {\sqrt {k\left( {k + 1} \right)\left( {k + 2} \right)\left( {k + 3} \right)} } \right\rfloor = \left\lfloor {\sqrt {t\left( {t + 2} \right)} } \right\rfloor = t$
$\Rightarrow U = \sum\limits_{k = 1}^n {\left\lfloor {\sqrt {k\left( {k + 1} \right)\left( {k + 2} \right)\left( {k + 3} \right)} } \right\rfloor } = \sum\limits_{k = 1}^n {k\left( {k + 3} \right)} = \dfrac{{5n^3 + 13n}}{8} $
"Do you still... believe in me ?" Sarah Kerrigan asked Jim Raynor - Starcraft II:Heart Of The Swarm.

#7
hxthanh

hxthanh

    Tín đồ $\sum$

  • Hiệp sỹ
  • 3921 Bài viết
Rồi, ok anh đã sửa! (Em xem lại kết quả của em nhé!)
Chém nốt bài 1 nhé

Ta có BĐT sau
$\dfrac{1}{\sqrt{k+1}}<2(\sqrt{k+1}-\sqrt{k})<\dfrac{1}{\sqrt {k}}$
Do đó cho k chay từ 1 đến n, ta có
$\dfrac{1}{\sqrt{n+1}}-1+\sum_{k=1}^n \dfrac{1}{\sqrt{k}}<2(\sqrt{n+1}-1)<\sum_{k=1}^n \dfrac{1}{\sqrt{k}}$

Suy ra

$2\sqrt{n+1}-2<H=\sum_{k=1}^n \dfrac{1}{\sqrt{k}}<2\sqrt{n+1}-1-\dfrac{1}{\sqrt{n+1}}\;\;(???)$

Đến đây cũng chưa thể kết luận được xem $\lfloor H\rfloor$ bằng bao nhiêu đâu dark templar ơi!
Nếu $H$ mà nguyên thì dễ rồi
Vì trong (???) hiệu $VP-VT<1$ nên nếu $H $ nguyên thì $H=\lfloor VP\rfloor$
Nhưng ở đây chắc chắn $H $ không nguyên. Do đó hoặc $\lfloor H\rfloor=\lfloor VT\rfloor$ hoặc là $\lfloor H\rfloor=\lfloor VP\rfloor$


Thử với $n=4 $, rồi $n=5$ là biết ngay!

$\bullet\;\;n=4 \Rightarrow VT \approx 2.47;\;\;H \approx 2.78;\;\;VP \approx 3.02$
$\bullet\;\;n=5 \Rightarrow VT \approx 2.89;\;\;H \approx 3.23;\;\;VP \approx 3.49$

Bài viết đã được chỉnh sửa nội dung bởi hxthanh: 13-02-2011 - 03:06


#8
dark templar

dark templar

    Kael-Invoker

  • Hiệp sỹ
  • 3788 Bài viết
Kết quả bài 1 đúng rồi đó anh :D
Còn bài 2 thì chắc lúc em tính pt sai phân của dãy $\left\{ {a_n } \right\}:a_{n + 1} = a_n + \left( {n + 1} \right)\left( {n + 4} \right),a_1 = 3$ bị nhầm chỗ nào đó ;)

Bài viết đã được chỉnh sửa nội dung bởi dark templar: 12-02-2011 - 20:49

"Do you still... believe in me ?" Sarah Kerrigan asked Jim Raynor - Starcraft II:Heart Of The Swarm.

#9
hxthanh

hxthanh

    Tín đồ $\sum$

  • Hiệp sỹ
  • 3921 Bài viết

Không bik e tính có đúng ko thử nhẩm qua thì số hạng tổng quát của bài này là:
a) $ u_n=\left\lceil {\dfrac{\sqrt{8n+1}-1}{2}} \right\rceil $
b) Nếu $ n=k^2 $ thì $ u_n=2k-1 $ còn nếu n ko là SCP thì $ u_n=\left\lceil {4n+1} \right\rceil -1 $

Cường làm không đúng rồi. Em thử trình bày ra xem nào!
Em chú ý hai hàm ceiling $\lceil\;\rceil$và floor $\lfloor\;\rfloor$ khác nhau đấy nhé!

#10
hxthanh

hxthanh

    Tín đồ $\sum$

  • Hiệp sỹ
  • 3921 Bài viết
À phải rồi, kết quả của em chính xác, tuy nhiên kết quả viết dưới dạng phần nguyên sẽ thân thiện hơn
$u_n=\left\lceil\dfrac{\sqrt{8n+1}-1}{2}\right\rceil=\left\lfloor\dfrac{\sqrt{8n-7}+1}{2}\right\rfloor$
Còn câu b) không đơn giản đâu :P

#11
hxthanh

hxthanh

    Tín đồ $\sum$

  • Hiệp sỹ
  • 3921 Bài viết
Bài 2
b$)\;\;\;\;\{u_n\}_1^\infty:\;\{1,2,3,3,4,4,5,5,5,6,6,6,7,...\}$

(1 số 1 & 1 số 2); (2 số 3 & 2 số 4); ... ;(k số 2k-1 & k số 2k); ...
----
Quy luật trên đã xác định 1 dãy duy nhất!
--------------------------
TH1: Nếu $u_n=2k-1$ ( thuộc nhóm số lẻ )
Khi đó có $k-1$ nhóm số hạng lẻ và $k-1$ nhóm số hạng chẵn nhỏ hơn $u_n$
Đếm các số hạng nhỏ hơn $u_n$ ta được:
$(1+...+k-1)_\text{so chan}+(1+...+k-1)_\text{so le}=k^2-k$, (số hạng)
Do đó chỉ số $ n $ phải thỏa mãn $k^2-k+1 \le n \le k^2$
$\Rightarrow \sqrt n \le k \le \sqrt{n-\dfrac{3}{4}}+\dfrac{1}{2}$
Vì $\sqrt n > \sqrt{n-\dfrac{3}{4}}-\dfrac{1}{2}$
Suy ra $k=\left\lfloor \sqrt{n-\dfrac{3}{4}}+\dfrac{1}{2}\right\rfloor\;\;\;(1)$

TH2: Nếu $u_n=2m$ (thuộc nhóm số chẵn)
Khi đó có $m$ nhóm số hạng lẻ và $m-1$ nhóm số hạng chẵn nhỏ hơn $u_n$
Đếm các số hạng nhỏ hơn $u_n$ ta được:
$(1+...+m-1)_\text{so chan}+(1+...+m)_\text{so le}=m^2$, (số hạng)
Do đó chỉ số $ n $ phải thỏa mãn $m^2+1 \le n \le m^2+m$
$\Rightarrow \sqrt{n+\dfrac{1}{4}}-\dfrac{1}{2} \le m \le \sqrt{n-1}$
Vì $\sqrt{n+\dfrac{1}{4}}-\dfrac{1}{2}>\sqrt{n-1}-1$
Suy ra $m=\left\lfloor \sqrt{n-1}\right\rfloor\;\;\;(2)$

Xét tổng $k+m\;\;\;(\text{theo}\;(1)\&(2))$ trong 2 trường hợp sau:
(#1) $i^2-i+1 \le n \le i^2$
TH này ta có: $u_n=2i-1,\;\;\;k=i$ và
$i-1=\left\lfloor\sqrt{i^2-i}\right\rfloor\le\left\lfloor\sqrt{n-1}\right\rfloor\le\left\lfloor\sqrt{i^2-1}\right\rfloor=i-1$
Do đó $k+m=2i-1=u_n$
(#2) $i^2+1 \le n \le i^2+i$
TH này ta có: $u_n=2i,\;\;\;m=i$ và
$i=\left\lfloor\sqrt{i^2+\dfrac{1}{4}}+\dfrac{1}{2}\right\rfloor\le\left\lfloor\sqrt{n^2-\dfrac{3}{4}}+\dfrac{1}{2}\right\rfloor\le\left\lfloor\sqrt{i^2+i-\dfrac{3}{4}}+\dfrac{1}{2}\right\rfloor=i$
Do đó cũng có $k+m=2i=u_n$

Tóm lại ta có công thức tổng quát của dãy $\{u_n\}$ là
$\boxed{u_n=\left\lfloor \sqrt{n-\dfrac{3}{4}}+\dfrac{1}{2}\right\rfloor+\left\lfloor \sqrt{n-1}\right\rfloor}$
hay (để nhìn cho đẹp @};- )
$\boxed{u_n=\left\lfloor \sqrt n +\dfrac{1}{2}\right\rfloor+\left\lfloor \sqrt{n-1}\right\rfloor}$ Tất nhiên cần phải CM $\left\lfloor \sqrt{n-\dfrac{3}{4}}+\dfrac{1}{2}\right\rfloor=\left\lfloor \sqrt n +\dfrac{1}{2}\right\rfloor$
------
Tính tổng n số hạng đầu tiên:
$S_n=\sum_{k=1}^n u_k = \sum_{k=1}^n \left\lfloor \sqrt k +\dfrac{1}{2}\right\rfloor +\sum_{k=1}^n \left\lfloor \sqrt{k-1}\right\rfloor $

dark templar hay bạn nào đó chém nốt cái!


#12
dark templar

dark templar

    Kael-Invoker

  • Hiệp sỹ
  • 3788 Bài viết

Bài 2
b$)\;\;\;\;\{u_n\}_1^\infty:\;\{1,2,3,3,4,4,5,5,5,6,6,6,7,...\}$

(1 số 1 & 1 số 2); (2 số 3 & 2 số 4); ... ;(k số 2k-1 & k số 2k); ...
----
Quy luật trên đã xác định 1 dãy duy nhất!
--------------------------

------
Tính tổng n số hạng đầu tiên:
$S_n=\sum_{k=1}^n u_k = \sum_{k=1}^n \left\lfloor \sqrt k +\dfrac{1}{2}\right\rfloor +\sum_{k=1}^n \left\lfloor \sqrt{k-1}\right\rfloor $

dark templar hay bạn nào đó chém nốt cái :D

Em nghĩ nếu anh để là nếu k là số chẵn thì sẽ có k số chẵn và k số lẻ ->có k-1 nhóm số chẵn và k-1 nhóm số lẻ thì sẽ dễ hiểu hơn đấy ạ ;)
Còn câu tính tổng thì ta sử dụng 1 trường hợp riêng của định lý Hermitte :
$S_n = \sum\limits_{k = 1}^n {u_k } = \sum\limits_{k = 1}^n {\left\lfloor {\sqrt k + \dfrac{1}{2}} \right\rfloor } + \sum\limits_{k = 1}^n {\left\lfloor {\sqrt {k - 1} } \right\rfloor } = \sum\limits_{k = 1}^n {\left\lfloor {2\sqrt k } \right\rfloor } - \sum\limits_{k = 1}^n {\left\lfloor {\sqrt k } \right\rfloor } + \sum\limits_{k = 1}^n {\left\lfloor {\sqrt {k - 1} } \right\rfloor }$
Đến đây thì 2 cái tổng đầu thì ta đã tính đc :perp(trong topic "Phần nguyên của anh chứ đâu ) Cái tổng thứ 3 thì cách tính cũng dựa vào tổng thứ 2 thôi :D
"Do you still... believe in me ?" Sarah Kerrigan asked Jim Raynor - Starcraft II:Heart Of The Swarm.

#13
hxthanh

hxthanh

    Tín đồ $\sum$

  • Hiệp sỹ
  • 3921 Bài viết

Em nghĩ nếu anh để là nếu k là số chẵn thì sẽ có k số chẵn và k số lẻ ->có k-1 nhóm số chẵn và k-1 nhóm số lẻ thì sẽ dễ hiểu hơn đấy ạ :D
Còn câu tính tổng thì ta sử dụng 1 trường hợp riêng của định lý Hermitte :
$S_n = \sum\limits_{k = 1}^n {u_k } = \sum\limits_{k = 1}^n {\left\lfloor {\sqrt k + \dfrac{1}{2}} \right\rfloor } + \sum\limits_{k = 1}^n {\left\lfloor {\sqrt {k - 1} } \right\rfloor } = \sum\limits_{k = 1}^n {\left\lfloor {2\sqrt k } \right\rfloor } - \sum\limits_{k = 1}^n {\left\lfloor {\sqrt k } \right\rfloor } + \sum\limits_{k = 1}^n {\left\lfloor {\sqrt {k - 1} } \right\rfloor }$
Đến đây thì 2 cái tổng đầu thì ta đã tính đc :perp(trong topic "Phần nguyên của anh chứ đâu ) Cái tổng thứ 3 thì cách tính cũng dựa vào tổng thứ 2 thôi :D

Suy luận của em thì đúng như ý tưởng của anh rồi (lộ đề ;) )
Em chú ý nhé! trong Topic [Phần Nguyên] là tổng này $\sum_{k=0}^n \left\lfloor \sqrt{2k}\right\rfloor$ cơ mà!
Một ý nữa là:
$\sum_{k=1}^n \left\lfloor \sqrt{k-1}\right\rfloor=\sum_{k=0}^{n-1} \left\lfloor \sqrt{k}\right\rfloor=\sum_{k=1}^n \left\lfloor \sqrt{k}\right\rfloor-\left\lfloor\sqrt{n}\right\rfloor$
Nên ta có:
$S_n=\sum_{k=1}^n \left\lfloor 2\sqrt{k}\right\rfloor-\left\lfloor \sqrt{n}\right\rfloor\;\;(\bullet)$
Mục đích của anh ở bài này là phải tính $S_n$ trực tiếp theo công thức tổng quát ban đầu.
Sau đó từ $(\bullet)$ ta sẽ suy ra được kết quả của:

Bài 3:
Tính tổng $S_n=\sum_{k=0}^n \left\lfloor 2\sqrt k\right\rfloor$

Em hiểu ý anh chứ ?!

------
Nếu không thì làm bài 3 trước rồi mới làm bài 2 :Rightarrow
Có ai làm được bài 4 không? - Làm được tôi thưởng cho... một tô mì tôm :D ăn đêm cho đỡ đói!

Bài viết đã được chỉnh sửa nội dung bởi hxthanh: 14-02-2011 - 14:35


#14
tuan101293

tuan101293

    Trung úy

  • Thành viên
  • 999 Bài viết
4,
Ta xét đồ thị $y=\sqrt[3]{px}$ với $1\le x\le (p-1)(p-2)$ ©
Đặt tổng đã cho là S,ta có số điểm nguyên dương nằm dưới đồ thị © chính là tổng S
Quay ngược trục tọa độ(tráo x cho y),ta có đồ thị © chính là $y=\dfrac{x^3}{p}$ với $1\le x\le \sqrt[3]{p(p-1)(p-2)}$
Số điểm nguyên nằm dưới đồ thị © bây h chính là tổng $S'=\sum [\dfrac{k^3}{p}]$ (chạy từ $1\le x\le [\sqrt[3]{p(p-1)(p-2)}]=p-2$)
nên ta có
$S+S'=(p-1)(p-2)^2$ (số điểm nguyên trong hình chữ nhật)
mà $S'=\dfrac{(p-1)(p-2)(p+1)}{4}-[\dfrac{(p-1)^3)}{p}]$
(tính S' đơn giản vì $[\dfrac{k^3}{p}]+[\dfrac{(p-k)^3}{p}]=\dfrac{k^3}{p}+\dfrac{(p-k)^3}{p}-1$)
nên ta có ngay ĐPCM
...
hy vọng được ăn mỳ tôm :)

Bài viết đã được chỉnh sửa nội dung bởi tuan101293: 14-02-2011 - 22:09

KT-PT


Do unto others as you would have them do unto you.


#15
hxthanh

hxthanh

    Tín đồ $\sum$

  • Hiệp sỹ
  • 3921 Bài viết
Ok, rồi cách đếm điểm nguyên là công cụ mạnh nhất rùi.
Cảm ơn Giáo sư!
Tuy nhiên còn 1 cách CM sơ cấp hơn :) nhưng hơi bị dài...
------
Giáo sư ăn mì hảo hảo hay oh ngon :) Còn mỗi nửa gói :))

#16
tuan101293

tuan101293

    Trung úy

  • Thành viên
  • 999 Bài viết
Em có thấy cách này trên mathlinks của pco.
cach 2 bai 4

KT-PT


Do unto others as you would have them do unto you.


#17
hxthanh

hxthanh

    Tín đồ $\sum$

  • Hiệp sỹ
  • 3921 Bài viết
Chuẩn!
Đó mới là cách anh muốn nói đến ở bài tập này
Chỗ này

...
Tính S' đơn giản vì $\left\lfloor\dfrac{k^3}{p}\right\rfloor+\left\lfloor\dfrac{(p-k)^3}{p}\right\rfloor=\dfrac{k^3}{p}+\dfrac{(p-k)^3}{p}-1$
...

là thế này:
Với $0<k<p$ mà $p$ nguyên tố thì $k^3$ không chia hết cho $p$

$\left\lfloor\dfrac{k^3}{p}\right\rfloor+ \left\lfloor \dfrac{(p-k)^3}{p}\right\rfloor=\left\lfloor\dfrac{k^3}{p}\right\rfloor+\left\lfloor\dfrac{-k^3}{p}\right\rfloor+\dfrac{(p-k)^3+k^3}{p}=-1+\dfrac{(p-k)^3+k^3}{p}$

(Sử dụng tính chất $\lfloor x\rfloor+\lfloor -x\rfloor=-1,\;\forall x \not\in \mathbb{Z}$)

Bài viết đã được chỉnh sửa nội dung bởi hxthanh: 21-02-2011 - 23:56


#18
hoangdang

hoangdang

    Thượng sĩ

  • Thành viên
  • 219 Bài viết

Chuẩn!
Đó mới là cách anh muốn nói đến ở bài tập này
Chỗ này

là thế này:
Với $0<k<p$ mà $p$ nguyên tố thì $k^3$ không chia hết cho $p$

$\left\lfloor\dfrac{k^3}{p}\right\rfloor+\left\lfloor\dfrac{(p-k)^3}{p}\right\rfloor=\left\lfloor\dfrac{k^3}{p}\right\rfloor+\left\lfloor\dfrac{-k^3}{p}\right\rfloor+\dfrac{(p-k)^3+k^3}{p}=-1+\dfrac{(p-k)^3+k^3}{p}$

(Sử dụng tính chất $\lfloor x\rfloor+\lfloor -x\rfloor=-1,\;\forall x \not\in \mathbb{Z}$)

cai nay may em THCS chem gay~ kiem

#19
hxthanh

hxthanh

    Tín đồ $\sum$

  • Hiệp sỹ
  • 3921 Bài viết

Anh hxthanh có thể viết 1 file pdf về mấy cái này và cả cái phần nguyên để cho anh em tiện theo dõi đc ko ạ =D>

Đây cũng là mong muốn của tôi, tài liệu sẽ gấp rút hoàn chỉnh trong nay mai.
Các bạn chịu khó chờ nhé!

#20
hxthanh

hxthanh

    Tín đồ $\sum$

  • Hiệp sỹ
  • 3921 Bài viết
@ tran_nguyen_quoc_cuong: File pdf về phần nguyên của em $\rightarrow$ ở đây $ \leftarrow $

Post thêm 2 bài:
$\boxed{\text{ 1 }}$ Cho dãy số $\{U_n\}$, xác định bởi: $U_1=1,\;\;\;U_2=3,\;\;\;U_3=4,\;\;\;U_{n+3}=U_n+4$
Tìm một công thức biểu diễn số hạng tổng quát $U_n$ theo $n$

$\boxed{\text{ 2 }}$ Cho dãy số $\{U_n\}_1^\infty$, xác định bằng quy luật lặp lại có chu kỳ như sau:

$\{U_n\}_1^\infty:\;\{1,2,2,3,3,3,4,5,5,6,6,6,7,8,...\}$

(1 số 1 | 2 số 2 | 3 số 3) ; (1 số 4 | 2 số 5 | 3 số 6) ; ....; (1 số 3k+1 | 2 số 3k+2 | 3 số 3k+3); ...

Tìm số hạng tổng quát của dãy số $\{U_n\}_1^\infty$ trên.





0 người đang xem chủ đề

0 thành viên, 0 khách, 0 thành viên ẩn danh